Conjecture très forte de Goldbach

2

Réponses

  • Vrfss
    Modifié (December 2022)
    Non la conjecture est la même juste que j'ai du mal à formuler mes propos, son résumé est je pense la meilleure version.
  • raoul.S
    Modifié (December 2022)
    gebrane a dit : 
    Ton message m'embrouille raoul, car dans sa conjecture comme tu l'as décrit, il suppose que Goldbach est vraie
    Non on ne suppose pas Goldbach. J'essaie autrement : on conjecture que si (c'est un si) un nombre pair $n$ est une somme de nombres premiers alors il existe un nombre premier $p$ et un nombre premier $q$ tel que $q+2$ est également premier et tels que $n=p+q$.

    À partir de là par récurrence tu as Goldbach et l'infinité des nombres premiers jumeaux en bonus :mrgreen:

    @Vrfss si c'est ça que tu voulais dire alors ok mais sincèrement je ne crois pas du tout que ça simplifie quoi que ce soit malheureusement...   
  • gerard0
    Modifié (December 2022)
    C'est sûr qu'en passant son temps à écrire des phrases floues, on finit par obtenir de quelqu'un d'autre une phrase utile. Donc on dira "la conjecture de Raoul.S".
  • Vrfss
    Modifié (December 2022)
    Oui @raoul.S, c'est bien là où je voulais en venir vu que j'ai bien appelé la discussion 2 conjectures en une, si on résout celle que j'ai énoncée on résout Goldbach et les nombres jumeaux en même temps ! 
  • Vrfss
    Modifié (December 2022)
    Bonsoir
    J'ai vérifié la conjecture forte de Goldbach (je pense qu'il est plus logique de l'appeler ainsi puisque ce n'est pas une découverte mais juste une restriction de la conjecture de Goldbach) pour 10^11 j'obtiens, 99999998939+1061.
    1061 étant bien jumeaux j'en déduis que 10^11+2= 99999998939+1063. Malheureusement je ne sais pas comment faire pour vérifier jusqu'à 10^11 si quelqu'un peut m'aider à orienter mes recherches étant donné que j'ai trouvé ces nombres sur un site qui donne les nombres premiers jusqu'à 10^11 et un peu plus donc le faire manuellement demanderait beaucoup de temps, je ne veux pas non plus abuser du temps de tout le monde libres à ceux qui le souhaitent de répondre.
    Cordialement.
  • Heuristique
    Modifié (December 2022)
    Bonjour
    La seconde conjecture sur les sommes de 2 nombres premiers jumeaux est fausse. Par exemple, 94 ne peut pas s'écrire comme somme de 2 nombres premiers jumeaux. Les nombres premiers jumeaux inférieurs à $100$ sont $3,5,7;11,13;17,19;29,31;41,43;59,61;71,73$ : il suffit de tester toutes les possibilités. On peut toutefois écrire $94 = 89 + 5$. $89$ n'est pas jumeau mais $5$ l'est.
    La première conjecture en revanche (tout nombre pair est somme d'un premier et d'un premier jumeau), se vérifie au moins jusqu'à $10^6$ (tu sauras qu'on écrit souvent un programme simple pour tester les conjectures sur des petits nombres, histoire d'éviter les contre-exemples évidents comme 94). Je n'ai pas cherché à optimiser mon programme donc je n'ai pas poussé plus loin les calculs. Je voulais juste savoir, vue la discussion, si un contre-exemple immédiat sortait. Apparemment pas.
  • Vrfss
    Modifié (December 2022)
    Oh d'accord je te remercie énormément d'avoir trouvé un contre-exemple à la deuxième conjecture (en soi c'était une idée qui m'est passé par la tête quand j'ai pensé à la première conjecture) et merci je vais essayer de trouver des programmes pour vérifier la première conjecture mais bon je dois avouer que je suis vraiment nul en programmation 😅
  • Une fois vérifiée jusqu'à un nombre raisonnable ($10^6$ est tout petit, mais c'est un bon début) on pourrait la baptiser conjecture ultra forte de Goldbach
    Il ne faut pas respirer la compote, ça fait tousser.

    J'affirme péremptoirement que toute affirmation péremptoire est fausse
  • Vrfss
    Modifié (December 2022)
    Je pense qu'il serait intéressant de se pencher sur cette conjecture selon moi puisque comme j'ai essayé (mais je me suis lamentablement vautré) de l'expliquer on peux essayer de mettre en place un raisonnement par récurrence qui pourrait être le début de la résolution de la conjecture de Goldbach, mais aussi de démontrer l'infinité de nombres premiers jumeaux par la même occasion dans le sens où cette conjecture serait vraie après je demande l'avis des gens qualifiés pour savoir si cette conjecture mériterait vraiment de l'attention :D 
  • Si elle est vérifiée jusqu'à $10^{18}$, par exemple, je dirais oui.
    Il ne faut pas respirer la compote, ça fait tousser.

    J'affirme péremptoirement que toute affirmation péremptoire est fausse
  • noobey
    Modifié (December 2022)
    @Vrfss
    Je pense savoir comment sauver ta conjecture, tu peux peut-être démontrer que tout entier pair est somme de 2 nombres premiers triplets (deux nombres premiers triplets c'est 2 nombres premiers espacés de 3, par exemple 2 et 5)

    Ça démontrerait à la fois la conjecture de Goldbach et la conjecture des nombres premiers triplets.
  • Vrfss
    Modifié (December 2022)
    @noobey d'après ce que j'ai compris tu proposerais une conjecture encore plus forte que la mienne étant donné que cette fois on démontrerait Goldbach, les premiers jumeaux ainsi que les premiers triplet ? (j'ai lu que si on démontre la conjecture des premiers triplet alors ça renforce la conjecture des premiers jumeaux après je me trompe peut-être).
  • Vous vous êtes fait piéger !
    Il ne faut pas respirer la compote, ça fait tousser.

    J'affirme péremptoirement que toute affirmation péremptoire est fausse
  • lourrran
    Modifié (December 2022)
    La conjecture semble vraie, quand on la vérifie jusqu'à $10^6$, et surtout, le nombre de décompositions de chaque nombre pair est relativement élevé.
    Pour les multiples de 6, on a plus de 1000 façons de décomposer le nombre en premier + premier-Jumeau, et pour les pairs non multiples de 6, on a plus de 500 décompositions.
    Donc comme dirait Goldbach, on constate que ça marche systématiquement, mais on n'a pas la moindre idée pour le démontrer.
    Tu me dis, j'oublie. Tu m'enseignes, je me souviens. Tu m'impliques, j'apprends. Benjamin Franklin
  • C'est vrai, on reste bloqué sur la question du pourquoi ça marche mais je trouve qu'on a quand même fait un tout petit pas en avant puisqu'on saurait démontrer pourquoi ça marche pour tout les paire sans en oublier un seul si la conjecture était vrai.
  • Conjecture : tous les nombres entiers sont inférieurs à $\pi^{314159^{1000}}$. 

    J’ai testé jusqu’à $10^{30}$. On sent bien que ça marche mais on ne sait pas pourquoi. 
    C’est ça, le problème des conjectures « qui marchent ». 
    Je le perds à chaque fois mais il existe un fil de discussion qui récence quelques conjectures… qui tombent pour des entiers « très très grands ». 
  • Les conjecture n'ont ni contre exemple ni démonstration ( enfin si il en existe juste qu'elle est très complexe si on l'a trouve pas) mais après (10^314159)^1000 est entier et plus grand non ?
  • Médiat_Suprème
    Modifié (December 2022)
    Cette conjecture est plus forte que Goldbach, donc, effectivement, on ne sait pas si elle est vraie ou fausse (c'est le propre d'une conjecture), néanmoins, si on peut la vérifier jusqu'à un nombre suffisamment grand, cela en ferait une conjecture digne d'intérêt, et nous la devons à Vrfss, semble-t-il. 

    Pour la prouver, il faut quand même, noter que cette conjecture entraîne Goldbach et les premiers jumeaux (le contraire n'est pas avéré), elle est de ce fait certainement plus difficile que chacune de ces deux là.
    Il ne faut pas respirer la compote, ça fait tousser.

    J'affirme péremptoirement que toute affirmation péremptoire est fausse
  • Je pense que cette conjecture n'a rien de nouveau.
    Si on réussit à déchiffrer les messages de LEG, je crois qu'il dit la même chose. 
    Tu me dis, j'oublie. Tu m'enseignes, je me souviens. Tu m'impliques, j'apprends. Benjamin Franklin
  • Médiat_Suprème
    Modifié (December 2022)
    N'étant pas Champollion ...
    La paternité n'est pas l'essentiel ici.
    Il ne faut pas respirer la compote, ça fait tousser.

    J'affirme péremptoirement que toute affirmation péremptoire est fausse
  • Vrfss
    Modifié (December 2022)
    Après pas de conclusion hâtive il faut vérifier que cette conjecture n'a pas déjà été émise auparavant (selon moi si quelqu'un l'avait déjà émise elle aurait suscité plus d'intérêt tout de même) peut-être a-t-elle déjà été émise mais à cause de sa difficulté oubliée.
  • Disons que cette conjecture a un intérêt très limité, car très redondante avec la conjecture de Goldbach.
    Sinon, allons-y, on doit pouvoir faire quelques dizaines de conjectures plus ou moins similaires : 
    - tout nombre pair supérieur à 10 peut s'écrire comme somme de 2 nombres premiers, dont l'un est le plus petit de 2 nombres premiers jumeaux.
    - tout nombre pair supérieur à 10 peut s'écrire comme somme de 2 nombres premiers, dont l'un est le plus grand de 2 nombres premiers jumeaux.
    - tout nombre pair supérieur à 10 peut s'écrire comme somme de 2 nombres premiers, dont l'un est le plus petit de 2 nombres premiers cousins.
    - tout nombre pair supérieur à 10 peut s'écrire comme somme de 2 nombres premiers, dont l'un est le plus grand  de 2 nombres premiers cousins.
    - tout nombre pair supérieur à 100 peut s'écrire comme somme d'un nombre premier jumeau et d'un nombre premier cousin. avec comme contrainte que le nombre premier jumeau soit plus petit que le nombre premier cousin
    - tout nombre pair supérieur à 100 peut s'écrire comme somme de 2 nombres premiers sexys
    etc etc 

    Je mets une restriction 'supérieur à 100' , parce que j'ai la flemme de vérifier pour les tout petits nombres. De toutes façons, avec la conjecture dont on parle, on a aussi une restriction (4 est exclu)
    Tu me dis, j'oublie. Tu m'enseignes, je me souviens. Tu m'impliques, j'apprends. Benjamin Franklin
  • Médiat_Suprème
    Modifié (December 2022)
    Il faudrait les vérifier jusqu'à $10^{18}$ ou même $10^{30}$
    Il ne faut pas respirer la compote, ça fait tousser.

    J'affirme péremptoirement que toute affirmation péremptoire est fausse
  • Vrfss
    Modifié (December 2022)
    C'est vrai, j'y ai pensé aussi à toute ces conjecture mais je trouve que on commence à avoir trop de restriction même si la véracité de ces conjecture pourrait démontrer l'infinité de nombres premiers cousins et sexy après moi je trouve les jumeaux ils sont bien par rapport au fait qu'on rajoute 2 à chaque fois et que pour passer d'un nombre pair au suivant on rajoute 2 aussi.
  • kioups
    Modifié (December 2022)
  • "la véracité de ces conjecture pourrait démontrer..."
    Le grand problème étant qu'on n'a aucun moyen de vérifier cette véracité !!
  • lourrran
    Modifié (December 2022)
    Les nombres jumeaux, c'est mieux, parce que pour passer de l'un à l'autre, on ajoute 2, et pour passer d'un nombre pair au suivant, on ajoute également 2.
    Soit, mais je dirais que c'est une illusion d'optique, un mirage. Ce n'est absolument pas une base pour faire une démonstration. J'ai vu passer le mot 'récurrence', aucun espoir de ce côté là.
    Distinguons les nombres jumeaux en 2 groupes, les $J_1$ et les $J_2$ : chaque couple $(J_1,J_2)$ est un couple de nombres premiers, tels que $J_2=J_1+2$.

    $41, 59$ sont des $J_1$, alors que $43$ et $61$ sont des $J_2$.
    Si on décompose un nombre pair $n$ avec un nombre premier et un nombre $J_1$, alors ok, $n+2$ sera décomposé automatiquement en remplaçant $J_1$ par $J_2$. La récurrence marche, mais elle s'arrête là.
    Tu me dis, j'oublie. Tu m'enseignes, je me souviens. Tu m'impliques, j'apprends. Benjamin Franklin
  • Vrfss
    Modifié (December 2022)
    C'est vrai, tu a raison, il faudrait prouver que la décomposition en P+J2 peux être remplacée par P'+J1 et seulement ainsi on pourrait montrer une récurrence pour tout n malheureusement de ce côté la démonstration bloque aussi.
  • Bonsoir, j'apporte une petite restriction a ma conjectures : P≥Q, si vous vous demandez l'intérêt et bien c'est pour éviter que ça marche avec les nombres premiers cousins ou sexy exemple : 12=7+5, 7 est cousin mais pas 5 ce qui fait que dans ce cas P≤C.
    Concernant la vérification de la conjecture avec des très grands nombres j'avance lentement mais sûrement il faut que je trouve des logiciels assez performant pour tester des très grands nombres je vous tiendrai informé.
    Cordialement 
  • Ta conjecture est très certainement vraie. Il n'y a pas trop de débat là dessus. Il y a des fluctuations, certains nombres ont plus de décompositions que qu'autres, mais on peut aisément les expliquer. On peut même très certainement bâtir une formule qui va estimer assez bien le nombre de décompositions pour chaque nombre pair.
    Le problème, c'est la démonstration. 
    En gros, la situation que tu proposes, c'est : La conjecture de Goldbach est excessivement compliquée à démontrer. On va donc tenter de démontrer une autre conjecture, encore plus difficile à démontrer.
    Tu me dis, j'oublie. Tu m'enseignes, je me souviens. Tu m'impliques, j'apprends. Benjamin Franklin
  • Vrfss
    Modifié (December 2022)
    Oui mon raisonnement est certainement très bizarre, pour résoudre un tel problème il faut bien explorer toute les solutions possibles, difficile ou facile, je doute guère que la conjecture de Goldbach puisse se démontrer en 3-4 lignes c'est pourquoi si on se penche sur quelques choses de plus complexe le résultat (si on y arrive) collerait sans doute plus à la réalité que des tentatives de démonstration qui tiennent en 3 ou 4 lignes après ce n'est pas à exclure que ce soit impossible mais je suis assez d'accord pour dire la probabilité est très faible.
  • Vrfss
    Modifié (December 2022)
    J'ai une question qui n'a pas trop de rapport mais je voulais éviter de créer une nouvelle discussion pour une simple question, y a-t-il une démonstration qui expliquerait que on peut trouver le nombre de manière de décomposer un nombre pair en somme de 2 nombres impairs en divisant le nombre pair par 4 et que si ce dernier n'est pas divisible par 4 on lui rajoute 2 ?
    Exemple : pour 16 on a 4 manières différentes de la décomposer en somme de 2 nombres impairs (1+15,3+13,5+11,9+7) et pour 10 on en a 3 (1+9,3+7,5+5).
    [Accord de genre : un nombre impair,   une somme paire. AD]  
  • Bibix
    Modifié (December 2022)
    Oui, $4 n = 2 (n + u) - 1 + 2(n - u) + 1$ pour tout $0 \leqslant u \leqslant n$. Et $4 n + 2 = 2 (n + u) + 1 + 2(n-u)+1$. Donc pour $4n$ et $4 n + 2$, il y a $n + 1$ décompositions.
  • Merci bien :)
  • ROBOUL75
    Modifié (November 2023)
    Bonjour
    La conjecture forte est démontrée, car selon Harald Helfgott, la conjecture faible ternaire annonce que tout nombre impair est la somme de trois nombre premiers, or (nombre pair - 1) = Somme de trois nombre premiers, nombre pair =  somme de trois nombres premiers + 1 , nombre pair = Somme de 2 premiers.
  • Math Coss
    Modifié (November 2023)
    Quel nul ce Helfgott ! Il fait des choses très compliquées et il ne se rend même pas compte qu'il démontre la conjecture forte ! Ou bien il y a un problème dans cet argument ?
    J'explicite : sachant décomposer $2n+1$ comme somme de trois nombres premiers, comment passer d'une décomposition $2n+1=p_1+p_2+p_3$ à une de la forme $2n=p_4+p_5$ ?
  • [Utilisateur supprimé]
    Modifié (November 2023)
    À mon avis, c'est jouable
    en gros a partir de
      
    Cela veux dire que quelle que soit la valeur, cette valeur et décomposable en somme $(2n)mod(p_n)=(0,,,,(p_n-1))= cts+n$ avec $n$ différent de zéro et $Syn[]_{cte}=P_j$ Pour les non-anglophones, à partir de $2n$, je calcule tous les modulo $(2n) mod (p_n)$ avec $p_n < 2n$ . Ensuite, je  décompose en sommes  les différente valeurs sans introduire de zéro, et après, je reconstitue les entiers . J'ai ainsi deux nombres premiers tels que $2n = p_a + p_b$ parce qu’il y a aucun zéro

  • @remy123456 : j'ai eu sensiblement la même idée que toi il y a une douzaine d'années. Si tu lis l'anglais et ne crains pas la migraine, tu peux toujours t'amuser à regarder la question intitulée "About Goldbach's conjecture" sur Mathoverflow, mais ça reste une conjecture, sauf au cinéma...
  • J'avais posté sur le site, mais leur politique assez élitiste a fait que je n'avais pas eu assez de votes.
  • ROBOUL75
    Modifié (November 2023)
    Pourtant ce n'est pas difficile à comprendre : Descartes savait que tout nombre pair  = sommes de deux nombres premiers 
    Harald Helgoff a prouvé la conjecture ternaire de Goldbach que tout nombre impair supérieur à 5 est la somme de trois nombres premiers
    cela équivaut  à (tout nombre pair + 1) = somme de trois nombres premiers 
    tout nombre pair  =  nombre premier  + nombre premier + nombre premier + 1 
    tout nombre pair  = nombre premier + nombre premier + nombre pair  
    tout nombre pair  = nombre premier  + nombre premier 
    tout nombre premier jumeaux = nombre pair  + nombre premier 
    donc nombre premier jumeaux  = nombre premier  + 2 A 
    conjecture de Riemann  = zeta => Sin /X = pi /3
    Somme (1/premier^s) = somme(1/2A^s) =pi/3 => Somme(1/2a^2S) = pi ^2 /6
    [Même dans shtam  Bernhard Riemann (1826-1866) prend toujours une majuscule. AD]
  • gerard0
    Modifié (November 2023)
    Quel méli-mélo !

    "Descartes savait que tout nombre pair  = sommes de deux nombres premiers " ??? Bizarre, la conjecture est celle de Goldbach, et elle n'est pas démontrée !

    "(tout nombre pair + 1) = somme de trois nombres premiers
    tout nombre pair  =  nombre premier  + nombre premier + nombre premier + 1
    tout nombre pair  = nombre premier + nombre premier + nombre pair" exact
    "tout nombre pair  = nombre premier  + nombre premier" Aucun lien avec ce qui précède !!
    C'est l'erreur fréquente des incompétents.

    tout nombre pair n = nombre premier qui dépend de n + nombre premier qui dépend de n + nombre pair p qui dépend de n
    Quand on soustrait p des deux côtés, on obtient
    n-p = nombre premier qui dépend de n + nombre premier qui dépend de n
    Mais n-p est un nombre pair particulier, qui dépend de n. Pas n'importe quel entier pair.

    Je passe sur la suite, qui ressemble plus à un mantra ou une formule cabalistique qu'à un texte sur les mathématiques.
    Même ici, les âneries restent des âneries.
  • LEG
    LEG
    Modifié (November 2023)
    Bonjour
    Sauf erreur il a été montré que si La conjecture de Goldbach est vraie , alors elle implique l'infinité de premiers jumeaux .

    Je suppose justement que c'est du fait, que pour toute décomposition de $2n > 6$ en la somme de deux nombres premiers $p+q$; il y a toujours parmi ces couples $p+q$ qui décomposent $2n$, un couple tel que $p$  appartient à un couple de jumeaux , ou $q$  ... 
    Même pour les nombres pairs qui ont moins de décompositions , par exemple les $2n$ de la forme $30k+2$ , $30k+4$ , $30k+8$ , $30k+14$ ...etc il y  a 8 formes.
     Mais pour chacune de ces formes , il y a toujours deux familles de premiers jumeaux et pour les $2n$ de la forme $30k+28$ les trois familles jumelles sont représentées . $30k + {11, 17 , 29}$ 

    Sachant que lorsque $2n$ tend vers l'infini le nombre de décompositions $p+q$ tend vers l'infini...
    Bon amusement
  • Sauf que pour Gerard0 pour sa gouverne, James Maynard a bien reçu une medaille field pour la conjecture de Goldmann forte pour tout nombre paire superieur à 2 est bien la somme de nombre premiers. Ou le comité de la médaille Fields est une armée d'ânes et Nobel avait raison d'en vouloir à l'Amant de sa femme car ce mathématicien l'a fait cocu, soit on nous cocufie à cause du Rsa et de la sécurité bancaire. Donc l'hypothèse forte de Goldman est résolue et donc les articles Prime in Tuple donnent les preuves des autres problèmes du millénaire et aussi ABC.
  • Quand on ne comprend rien à rien, il vaut mieux se taire...
  • gerard0
    Modifié (November 2023)
    Quand on fait référence à Nobel pour défendre sa "démonstration" on montre qu'on ne fait pas des maths ! Et qu'on ne comprend rien à rien, comme dit Poirot.
    Et quand on prétend que son raisonnement (faux !) démontre non seulement ce dont il parle, mais  aussi plusieurs conjectures célèbres qu'on associe indument, on montre son manque de connaissances et de compréhension, et son choix de prendre pour argent comptant ce que racontent les hurluberlus qui font de même, plutôt que d'accepter que la logique et les maths ne sont pas du baratin, mais des disciplines difficiles, qui demandent d'utiliser son intelligence.
    Roboul, tu peux faire illusion auprès d'incompétents, pas dans un forum de maths.
  • Chaurien
    Modifié (November 2023)
    On ne peut qu'approuver cette phrase de Poirot. 
    Moi j'aime bien la conjecture de Goldbach, que je trouve plus belle que la ci-devant de Fermat parce que celle de Goldbach est positive, en un certain sens. 
    Il y a dans l'Université des chercheurs en Théorie des Nombres, dont la qualification est reconnue. C'est à eux de dire quel est l'état actuel des connaissances relativement à telle ou telle conjecture relevant de ce domaine. Et d'ailleurs j'aimerais savoir où l'on en est pour celle-ci, ce serait plus intéressant que les élucubrations  qu'on lit ici.
    Il me semble peu probable, je veux dire fort improbable, que tel ou tel zozo trouve miraculeusement la réponse à une question que se posent tous les mathématiciens du monde, notamment les plus doués, depuis des siècles. Si par extraordinaire un chercheur isolé faisait une telle découverte, il pourrait la valider auprès des spécialistes universitaires qualifiés.
    C'est pourquoi je ne regarde jamais ce sous-forum « shtam » (!), c'est le nom de Goldbach qui m'y a attiré ce soir, mais je n'ai pas lu le fil en détail, et je salue encore la forte parole de Poirot, qui est tout ce qu'il y a à lire d'intéressant ici. Des forumeurs de qualité sont trop gentils de  perdre leur temps à répondre à des bêtises.  Et pour tout dire, je pense que notre forum serait plus sérieux s'il supprimait ce sous-forum ridicule qui n'est d'aucune utilité pour ceux qui s'intéressent vraiment aux mathématiques.
    Bonne soirée.
    Fr. Ch.
  • Bonjour, effectivement on est dans shtam, Roboul75 parle de conjecture de Goldman(n) au lieu de Goldbach et Chaurien ne le fait même pas remarquer, le monde va mal dans ce sous-forum. Blague à part, pour une fois, je suis d'accord avec lui.
    Histoire de ne pas poster pour rien, voici un exposé sur les travaux de James Maynard https://culturemath.ens.fr/sites/default/files/2023-05/Maynard_presentation_short.pdf
    In mémoriam de tous les professeurs assassinés dans l'exercice de leurs fonctions en 2023, n'oublions jamais les noms de Agnes-Lassalle et Dominique-Bernard qui n'ont pas donné lieu aux mêmes réactions sur ce forum (et merci à GaBuZoMeu)
  • S0_
    S0_
    Modifié (November 2023)
    Bonjour à tous,
    Moi je voudrais simplement proposer de directement aller à Celui qui parle mal que de proposer la fermeture du sous-forum...
    Bon c'est mon point de vu.
    Cordialement
    Bonaventure-S0_
  • LEG
    LEG
    Modifié (November 2023)
    Bonjour
    Mr @Chaurien : Ils se pourrait qu'il n'existe pas de démonstration rigoureuse de cette conjecture , ni de son indécidabilité ...

    Car quand même, à ce jour, personne n'a pu prouver qu'il existe effectivement une démonstration Mathématique rigoureuse de cette conjecture ...
    Alors que l'on a des ""preuves"" qu'elle ne peut pas être fausse à partir d'un entiers $2N$ assez grand ... sans pour autant être très grand ...

    On sait , que le nombre de décompositions d'un entier pair $2N > 4$ tend approximativement vers l'infini, lorsque $2N$ tend vers l'infini.

    On peut même conjecturer plus fort : pour tout $2N \geqslant{300} $ il existe une décomposition  en somme de deux nombres premiers $p+q$; avec une et seulement une seule famille de nombres premiers $P\leqslant N$ de la forme $30k+i$ avec $i\in{(1,7,11,13,17,19,23,29)}$  avec $q\in[N;2N]$ pas nécessairement de la même famille $i$ ...

    Voici un petit exemple pour $N = 6 600 000 000 010$ et les 4 familles $i$ de nombres premiers $p$ qui donnent le nombre de décompositions pour $2N = 13200000000020$ en somme de deux nombres premiers $p+q$ ... Avec $N$ qui augmente de $15$ ce qui permet de se faire une idée de la variation du nombre de décompositions de $2N$ par Famille $30k+i$ ; au lieu de tout mélanger ...et d'imaginer n'importe quoi .!
  • Chaurien
    Modifié (November 2023)
    Merci Vassillia pour ce document qui fait le point sur l'état des connaissances à propos des quatre problèmes de Landau, par quelqu'un qui sait de quoi il parle. Un îlot de sérieux dans un océan de bêtise...
    Maintenant j'aimerais un texte plus suivi que cet aperçu, avec bibliographie et historique détaillé pour chacun des problèmes. 
    Par exemple, je me souviens qu'il y a cinquante ans, nous avions salué la découverte de Chen Jingrun et nous avions espéré que c'était l'avant-dernière étape avant la démonstration de la conjecture de Goldbach :  https://en.wikipedia.org/wiki/Chen's_theorem.
    Chen était venu à Paris faire un exposé à un séminaire, et j'y avais assisté. Détail amusant : il était accompagné d'un domestique (pourrait-on dire) qui effaçait son tableau au fur et à mesure, et c'était en pleine époque maoîste...
    Bonne journée.
    Fr. Ch.
Connectez-vous ou Inscrivez-vous pour répondre.